Let $f$ be a continuous function on $[0,infty)$ such that $lim_{xto infty}(f(x)+int_0^x f(t)dt)$ exists. Find...












1














Let $f$ be a continuous function on $[0,infty)$ such that $lim_{xto infty}(f(x)+int_0^x f(t)dt)$ exists. Find $lim_{xto infty}f(x)$.



Useful hints will work. Please give some nice hint to solve the problem.










share|cite|improve this question
























  • Try taking $g=f+K$ with $K$ some constant. What happends? Can you guess the value of the limit?
    – xavierm02
    Feb 2 '17 at 14:36










  • @xavierm02 taking $g$=$f+K$ we will get $lim_{xto infty} f(x)+K$=$lim_{xto infty}g(x)$.Is it correct ? If yes whats next?
    – Piyush Raut
    Feb 2 '17 at 14:39












  • But what is $lim_{xto infty}(g(x)+int_0^x g(t)dt)$?
    – xavierm02
    Feb 2 '17 at 14:42












  • I didn't get that.Did you mean $lim_{xto infty}(f(x)+int_0^x f(t)dt)$?
    – Piyush Raut
    Feb 2 '17 at 14:45










  • No I meant with $g$, under the assumption that the limit exists with $f$.
    – xavierm02
    Feb 2 '17 at 14:45
















1














Let $f$ be a continuous function on $[0,infty)$ such that $lim_{xto infty}(f(x)+int_0^x f(t)dt)$ exists. Find $lim_{xto infty}f(x)$.



Useful hints will work. Please give some nice hint to solve the problem.










share|cite|improve this question
























  • Try taking $g=f+K$ with $K$ some constant. What happends? Can you guess the value of the limit?
    – xavierm02
    Feb 2 '17 at 14:36










  • @xavierm02 taking $g$=$f+K$ we will get $lim_{xto infty} f(x)+K$=$lim_{xto infty}g(x)$.Is it correct ? If yes whats next?
    – Piyush Raut
    Feb 2 '17 at 14:39












  • But what is $lim_{xto infty}(g(x)+int_0^x g(t)dt)$?
    – xavierm02
    Feb 2 '17 at 14:42












  • I didn't get that.Did you mean $lim_{xto infty}(f(x)+int_0^x f(t)dt)$?
    – Piyush Raut
    Feb 2 '17 at 14:45










  • No I meant with $g$, under the assumption that the limit exists with $f$.
    – xavierm02
    Feb 2 '17 at 14:45














1












1








1


1





Let $f$ be a continuous function on $[0,infty)$ such that $lim_{xto infty}(f(x)+int_0^x f(t)dt)$ exists. Find $lim_{xto infty}f(x)$.



Useful hints will work. Please give some nice hint to solve the problem.










share|cite|improve this question















Let $f$ be a continuous function on $[0,infty)$ such that $lim_{xto infty}(f(x)+int_0^x f(t)dt)$ exists. Find $lim_{xto infty}f(x)$.



Useful hints will work. Please give some nice hint to solve the problem.







integration limits functions continuity






share|cite|improve this question















share|cite|improve this question













share|cite|improve this question




share|cite|improve this question








edited Nov 29 '18 at 21:00









Shaun

8,820113681




8,820113681










asked Feb 2 '17 at 14:32









Piyush RautPiyush Raut

577




577












  • Try taking $g=f+K$ with $K$ some constant. What happends? Can you guess the value of the limit?
    – xavierm02
    Feb 2 '17 at 14:36










  • @xavierm02 taking $g$=$f+K$ we will get $lim_{xto infty} f(x)+K$=$lim_{xto infty}g(x)$.Is it correct ? If yes whats next?
    – Piyush Raut
    Feb 2 '17 at 14:39












  • But what is $lim_{xto infty}(g(x)+int_0^x g(t)dt)$?
    – xavierm02
    Feb 2 '17 at 14:42












  • I didn't get that.Did you mean $lim_{xto infty}(f(x)+int_0^x f(t)dt)$?
    – Piyush Raut
    Feb 2 '17 at 14:45










  • No I meant with $g$, under the assumption that the limit exists with $f$.
    – xavierm02
    Feb 2 '17 at 14:45


















  • Try taking $g=f+K$ with $K$ some constant. What happends? Can you guess the value of the limit?
    – xavierm02
    Feb 2 '17 at 14:36










  • @xavierm02 taking $g$=$f+K$ we will get $lim_{xto infty} f(x)+K$=$lim_{xto infty}g(x)$.Is it correct ? If yes whats next?
    – Piyush Raut
    Feb 2 '17 at 14:39












  • But what is $lim_{xto infty}(g(x)+int_0^x g(t)dt)$?
    – xavierm02
    Feb 2 '17 at 14:42












  • I didn't get that.Did you mean $lim_{xto infty}(f(x)+int_0^x f(t)dt)$?
    – Piyush Raut
    Feb 2 '17 at 14:45










  • No I meant with $g$, under the assumption that the limit exists with $f$.
    – xavierm02
    Feb 2 '17 at 14:45
















Try taking $g=f+K$ with $K$ some constant. What happends? Can you guess the value of the limit?
– xavierm02
Feb 2 '17 at 14:36




Try taking $g=f+K$ with $K$ some constant. What happends? Can you guess the value of the limit?
– xavierm02
Feb 2 '17 at 14:36












@xavierm02 taking $g$=$f+K$ we will get $lim_{xto infty} f(x)+K$=$lim_{xto infty}g(x)$.Is it correct ? If yes whats next?
– Piyush Raut
Feb 2 '17 at 14:39






@xavierm02 taking $g$=$f+K$ we will get $lim_{xto infty} f(x)+K$=$lim_{xto infty}g(x)$.Is it correct ? If yes whats next?
– Piyush Raut
Feb 2 '17 at 14:39














But what is $lim_{xto infty}(g(x)+int_0^x g(t)dt)$?
– xavierm02
Feb 2 '17 at 14:42






But what is $lim_{xto infty}(g(x)+int_0^x g(t)dt)$?
– xavierm02
Feb 2 '17 at 14:42














I didn't get that.Did you mean $lim_{xto infty}(f(x)+int_0^x f(t)dt)$?
– Piyush Raut
Feb 2 '17 at 14:45




I didn't get that.Did you mean $lim_{xto infty}(f(x)+int_0^x f(t)dt)$?
– Piyush Raut
Feb 2 '17 at 14:45












No I meant with $g$, under the assumption that the limit exists with $f$.
– xavierm02
Feb 2 '17 at 14:45




No I meant with $g$, under the assumption that the limit exists with $f$.
– xavierm02
Feb 2 '17 at 14:45










2 Answers
2






active

oldest

votes


















4














Let $F(x) = int_0^x f(t)dt$. Then, we have $lim_{xto infty} e^{-x}[e^x F(x)]' = c$ exists. From this, given $epsilon>0$, there is $N>0$ such that for all $x>N$,
$$
(c-epsilon)e^x leq[e^x F(x)]' leq (c+epsilon)e^x.
$$

Now, integrating from $N$ to $t$ gives:
$$(c-epsilon)(e^t-e^N)leq e^tF(t) -e^NF(N) leq (c+epsilon)(e^t-e^N),quadforall t>N
$$
and hence
$$
(c-epsilon)(1-e^{N-t})leq F(t) -e^{N-t}F(N) leq (c+epsilon)(1-e^{N-t}), quad forall t>N.
$$
Taking $ttoinfty$ yields
$$c-epsilon leq liminf_{tto infty}F(t)leq limsup_{tto infty}F(t)leq c+epsilon.
$$
Since $epsilon>0$ was arbitrary, we have $lim_{ttoinfty} F(t) =c$ and thus $lim_{ttoinfty} f(t) = 0$.






share|cite|improve this answer





























    3





    +50









    One of the easiest approaches is the use of L'Hospital's Rule here. Let $F(x) =int_{0}^{x}f(t),dt$ so that $F'(x) =f(x) $ via Fundamental Theorem of Calculus. We are thus given that $F(x) +F'(x) to L$ as $xtoinfty$.



    We have
    begin{align}
    lim_{xtoinfty} F(x) &=lim_{xtoinfty} frac{e^xF(x)} {e^x}notag\
    &=lim_{xto infty} dfrac{e^x{F(x) +F'(x) }} {e^x}text{ (via L'Hospital's Rule)} notag\
    &=lim_{xtoinfty} F(x) +F'(x)notag\
    &=Lnotag
    end{align}

    and hence $$lim_{xto infty} f(x) =lim_{xtoinfty} F'(x) =lim_{xtoinfty} {F(x) + F'(x) }-F(x) =L-L=0$$






    share|cite|improve this answer

















    • 1




      Nice solution, +1. But the flow is weird because you write $lim$ when it hasn't been shown the limit exists. Also, you don't know the limit of the numerator, $e^xF(x),$ is $infty.$ Now to use L'Hopital in this case, you don't need to know that. But many readers may not know this little fact.
      – zhw.
      Nov 30 '18 at 17:51












    • @zhw : I should have added more details. Thanks for putting these details in your comment.
      – Paramanand Singh
      Nov 30 '18 at 17:55











    Your Answer





    StackExchange.ifUsing("editor", function () {
    return StackExchange.using("mathjaxEditing", function () {
    StackExchange.MarkdownEditor.creationCallbacks.add(function (editor, postfix) {
    StackExchange.mathjaxEditing.prepareWmdForMathJax(editor, postfix, [["$", "$"], ["\\(","\\)"]]);
    });
    });
    }, "mathjax-editing");

    StackExchange.ready(function() {
    var channelOptions = {
    tags: "".split(" "),
    id: "69"
    };
    initTagRenderer("".split(" "), "".split(" "), channelOptions);

    StackExchange.using("externalEditor", function() {
    // Have to fire editor after snippets, if snippets enabled
    if (StackExchange.settings.snippets.snippetsEnabled) {
    StackExchange.using("snippets", function() {
    createEditor();
    });
    }
    else {
    createEditor();
    }
    });

    function createEditor() {
    StackExchange.prepareEditor({
    heartbeatType: 'answer',
    autoActivateHeartbeat: false,
    convertImagesToLinks: true,
    noModals: true,
    showLowRepImageUploadWarning: true,
    reputationToPostImages: 10,
    bindNavPrevention: true,
    postfix: "",
    imageUploader: {
    brandingHtml: "Powered by u003ca class="icon-imgur-white" href="https://imgur.com/"u003eu003c/au003e",
    contentPolicyHtml: "User contributions licensed under u003ca href="https://creativecommons.org/licenses/by-sa/3.0/"u003ecc by-sa 3.0 with attribution requiredu003c/au003e u003ca href="https://stackoverflow.com/legal/content-policy"u003e(content policy)u003c/au003e",
    allowUrls: true
    },
    noCode: true, onDemand: true,
    discardSelector: ".discard-answer"
    ,immediatelyShowMarkdownHelp:true
    });


    }
    });














    draft saved

    draft discarded


















    StackExchange.ready(
    function () {
    StackExchange.openid.initPostLogin('.new-post-login', 'https%3a%2f%2fmath.stackexchange.com%2fquestions%2f2125798%2flet-f-be-a-continuous-function-on-0-infty-such-that-lim-x-to-inftyf%23new-answer', 'question_page');
    }
    );

    Post as a guest















    Required, but never shown

























    2 Answers
    2






    active

    oldest

    votes








    2 Answers
    2






    active

    oldest

    votes









    active

    oldest

    votes






    active

    oldest

    votes









    4














    Let $F(x) = int_0^x f(t)dt$. Then, we have $lim_{xto infty} e^{-x}[e^x F(x)]' = c$ exists. From this, given $epsilon>0$, there is $N>0$ such that for all $x>N$,
    $$
    (c-epsilon)e^x leq[e^x F(x)]' leq (c+epsilon)e^x.
    $$

    Now, integrating from $N$ to $t$ gives:
    $$(c-epsilon)(e^t-e^N)leq e^tF(t) -e^NF(N) leq (c+epsilon)(e^t-e^N),quadforall t>N
    $$
    and hence
    $$
    (c-epsilon)(1-e^{N-t})leq F(t) -e^{N-t}F(N) leq (c+epsilon)(1-e^{N-t}), quad forall t>N.
    $$
    Taking $ttoinfty$ yields
    $$c-epsilon leq liminf_{tto infty}F(t)leq limsup_{tto infty}F(t)leq c+epsilon.
    $$
    Since $epsilon>0$ was arbitrary, we have $lim_{ttoinfty} F(t) =c$ and thus $lim_{ttoinfty} f(t) = 0$.






    share|cite|improve this answer


























      4














      Let $F(x) = int_0^x f(t)dt$. Then, we have $lim_{xto infty} e^{-x}[e^x F(x)]' = c$ exists. From this, given $epsilon>0$, there is $N>0$ such that for all $x>N$,
      $$
      (c-epsilon)e^x leq[e^x F(x)]' leq (c+epsilon)e^x.
      $$

      Now, integrating from $N$ to $t$ gives:
      $$(c-epsilon)(e^t-e^N)leq e^tF(t) -e^NF(N) leq (c+epsilon)(e^t-e^N),quadforall t>N
      $$
      and hence
      $$
      (c-epsilon)(1-e^{N-t})leq F(t) -e^{N-t}F(N) leq (c+epsilon)(1-e^{N-t}), quad forall t>N.
      $$
      Taking $ttoinfty$ yields
      $$c-epsilon leq liminf_{tto infty}F(t)leq limsup_{tto infty}F(t)leq c+epsilon.
      $$
      Since $epsilon>0$ was arbitrary, we have $lim_{ttoinfty} F(t) =c$ and thus $lim_{ttoinfty} f(t) = 0$.






      share|cite|improve this answer
























        4












        4








        4






        Let $F(x) = int_0^x f(t)dt$. Then, we have $lim_{xto infty} e^{-x}[e^x F(x)]' = c$ exists. From this, given $epsilon>0$, there is $N>0$ such that for all $x>N$,
        $$
        (c-epsilon)e^x leq[e^x F(x)]' leq (c+epsilon)e^x.
        $$

        Now, integrating from $N$ to $t$ gives:
        $$(c-epsilon)(e^t-e^N)leq e^tF(t) -e^NF(N) leq (c+epsilon)(e^t-e^N),quadforall t>N
        $$
        and hence
        $$
        (c-epsilon)(1-e^{N-t})leq F(t) -e^{N-t}F(N) leq (c+epsilon)(1-e^{N-t}), quad forall t>N.
        $$
        Taking $ttoinfty$ yields
        $$c-epsilon leq liminf_{tto infty}F(t)leq limsup_{tto infty}F(t)leq c+epsilon.
        $$
        Since $epsilon>0$ was arbitrary, we have $lim_{ttoinfty} F(t) =c$ and thus $lim_{ttoinfty} f(t) = 0$.






        share|cite|improve this answer












        Let $F(x) = int_0^x f(t)dt$. Then, we have $lim_{xto infty} e^{-x}[e^x F(x)]' = c$ exists. From this, given $epsilon>0$, there is $N>0$ such that for all $x>N$,
        $$
        (c-epsilon)e^x leq[e^x F(x)]' leq (c+epsilon)e^x.
        $$

        Now, integrating from $N$ to $t$ gives:
        $$(c-epsilon)(e^t-e^N)leq e^tF(t) -e^NF(N) leq (c+epsilon)(e^t-e^N),quadforall t>N
        $$
        and hence
        $$
        (c-epsilon)(1-e^{N-t})leq F(t) -e^{N-t}F(N) leq (c+epsilon)(1-e^{N-t}), quad forall t>N.
        $$
        Taking $ttoinfty$ yields
        $$c-epsilon leq liminf_{tto infty}F(t)leq limsup_{tto infty}F(t)leq c+epsilon.
        $$
        Since $epsilon>0$ was arbitrary, we have $lim_{ttoinfty} F(t) =c$ and thus $lim_{ttoinfty} f(t) = 0$.







        share|cite|improve this answer












        share|cite|improve this answer



        share|cite|improve this answer










        answered Nov 27 '18 at 7:45









        SongSong

        6,948421




        6,948421























            3





            +50









            One of the easiest approaches is the use of L'Hospital's Rule here. Let $F(x) =int_{0}^{x}f(t),dt$ so that $F'(x) =f(x) $ via Fundamental Theorem of Calculus. We are thus given that $F(x) +F'(x) to L$ as $xtoinfty$.



            We have
            begin{align}
            lim_{xtoinfty} F(x) &=lim_{xtoinfty} frac{e^xF(x)} {e^x}notag\
            &=lim_{xto infty} dfrac{e^x{F(x) +F'(x) }} {e^x}text{ (via L'Hospital's Rule)} notag\
            &=lim_{xtoinfty} F(x) +F'(x)notag\
            &=Lnotag
            end{align}

            and hence $$lim_{xto infty} f(x) =lim_{xtoinfty} F'(x) =lim_{xtoinfty} {F(x) + F'(x) }-F(x) =L-L=0$$






            share|cite|improve this answer

















            • 1




              Nice solution, +1. But the flow is weird because you write $lim$ when it hasn't been shown the limit exists. Also, you don't know the limit of the numerator, $e^xF(x),$ is $infty.$ Now to use L'Hopital in this case, you don't need to know that. But many readers may not know this little fact.
              – zhw.
              Nov 30 '18 at 17:51












            • @zhw : I should have added more details. Thanks for putting these details in your comment.
              – Paramanand Singh
              Nov 30 '18 at 17:55
















            3





            +50









            One of the easiest approaches is the use of L'Hospital's Rule here. Let $F(x) =int_{0}^{x}f(t),dt$ so that $F'(x) =f(x) $ via Fundamental Theorem of Calculus. We are thus given that $F(x) +F'(x) to L$ as $xtoinfty$.



            We have
            begin{align}
            lim_{xtoinfty} F(x) &=lim_{xtoinfty} frac{e^xF(x)} {e^x}notag\
            &=lim_{xto infty} dfrac{e^x{F(x) +F'(x) }} {e^x}text{ (via L'Hospital's Rule)} notag\
            &=lim_{xtoinfty} F(x) +F'(x)notag\
            &=Lnotag
            end{align}

            and hence $$lim_{xto infty} f(x) =lim_{xtoinfty} F'(x) =lim_{xtoinfty} {F(x) + F'(x) }-F(x) =L-L=0$$






            share|cite|improve this answer

















            • 1




              Nice solution, +1. But the flow is weird because you write $lim$ when it hasn't been shown the limit exists. Also, you don't know the limit of the numerator, $e^xF(x),$ is $infty.$ Now to use L'Hopital in this case, you don't need to know that. But many readers may not know this little fact.
              – zhw.
              Nov 30 '18 at 17:51












            • @zhw : I should have added more details. Thanks for putting these details in your comment.
              – Paramanand Singh
              Nov 30 '18 at 17:55














            3





            +50







            3





            +50



            3




            +50




            One of the easiest approaches is the use of L'Hospital's Rule here. Let $F(x) =int_{0}^{x}f(t),dt$ so that $F'(x) =f(x) $ via Fundamental Theorem of Calculus. We are thus given that $F(x) +F'(x) to L$ as $xtoinfty$.



            We have
            begin{align}
            lim_{xtoinfty} F(x) &=lim_{xtoinfty} frac{e^xF(x)} {e^x}notag\
            &=lim_{xto infty} dfrac{e^x{F(x) +F'(x) }} {e^x}text{ (via L'Hospital's Rule)} notag\
            &=lim_{xtoinfty} F(x) +F'(x)notag\
            &=Lnotag
            end{align}

            and hence $$lim_{xto infty} f(x) =lim_{xtoinfty} F'(x) =lim_{xtoinfty} {F(x) + F'(x) }-F(x) =L-L=0$$






            share|cite|improve this answer












            One of the easiest approaches is the use of L'Hospital's Rule here. Let $F(x) =int_{0}^{x}f(t),dt$ so that $F'(x) =f(x) $ via Fundamental Theorem of Calculus. We are thus given that $F(x) +F'(x) to L$ as $xtoinfty$.



            We have
            begin{align}
            lim_{xtoinfty} F(x) &=lim_{xtoinfty} frac{e^xF(x)} {e^x}notag\
            &=lim_{xto infty} dfrac{e^x{F(x) +F'(x) }} {e^x}text{ (via L'Hospital's Rule)} notag\
            &=lim_{xtoinfty} F(x) +F'(x)notag\
            &=Lnotag
            end{align}

            and hence $$lim_{xto infty} f(x) =lim_{xtoinfty} F'(x) =lim_{xtoinfty} {F(x) + F'(x) }-F(x) =L-L=0$$







            share|cite|improve this answer












            share|cite|improve this answer



            share|cite|improve this answer










            answered Nov 30 '18 at 2:37









            Paramanand SinghParamanand Singh

            49.2k555161




            49.2k555161








            • 1




              Nice solution, +1. But the flow is weird because you write $lim$ when it hasn't been shown the limit exists. Also, you don't know the limit of the numerator, $e^xF(x),$ is $infty.$ Now to use L'Hopital in this case, you don't need to know that. But many readers may not know this little fact.
              – zhw.
              Nov 30 '18 at 17:51












            • @zhw : I should have added more details. Thanks for putting these details in your comment.
              – Paramanand Singh
              Nov 30 '18 at 17:55














            • 1




              Nice solution, +1. But the flow is weird because you write $lim$ when it hasn't been shown the limit exists. Also, you don't know the limit of the numerator, $e^xF(x),$ is $infty.$ Now to use L'Hopital in this case, you don't need to know that. But many readers may not know this little fact.
              – zhw.
              Nov 30 '18 at 17:51












            • @zhw : I should have added more details. Thanks for putting these details in your comment.
              – Paramanand Singh
              Nov 30 '18 at 17:55








            1




            1




            Nice solution, +1. But the flow is weird because you write $lim$ when it hasn't been shown the limit exists. Also, you don't know the limit of the numerator, $e^xF(x),$ is $infty.$ Now to use L'Hopital in this case, you don't need to know that. But many readers may not know this little fact.
            – zhw.
            Nov 30 '18 at 17:51






            Nice solution, +1. But the flow is weird because you write $lim$ when it hasn't been shown the limit exists. Also, you don't know the limit of the numerator, $e^xF(x),$ is $infty.$ Now to use L'Hopital in this case, you don't need to know that. But many readers may not know this little fact.
            – zhw.
            Nov 30 '18 at 17:51














            @zhw : I should have added more details. Thanks for putting these details in your comment.
            – Paramanand Singh
            Nov 30 '18 at 17:55




            @zhw : I should have added more details. Thanks for putting these details in your comment.
            – Paramanand Singh
            Nov 30 '18 at 17:55


















            draft saved

            draft discarded




















































            Thanks for contributing an answer to Mathematics Stack Exchange!


            • Please be sure to answer the question. Provide details and share your research!

            But avoid



            • Asking for help, clarification, or responding to other answers.

            • Making statements based on opinion; back them up with references or personal experience.


            Use MathJax to format equations. MathJax reference.


            To learn more, see our tips on writing great answers.




            draft saved


            draft discarded














            StackExchange.ready(
            function () {
            StackExchange.openid.initPostLogin('.new-post-login', 'https%3a%2f%2fmath.stackexchange.com%2fquestions%2f2125798%2flet-f-be-a-continuous-function-on-0-infty-such-that-lim-x-to-inftyf%23new-answer', 'question_page');
            }
            );

            Post as a guest















            Required, but never shown





















































            Required, but never shown














            Required, but never shown












            Required, but never shown







            Required, but never shown

































            Required, but never shown














            Required, but never shown












            Required, but never shown







            Required, but never shown







            Popular posts from this blog

            Quarter-circle Tiles

            build a pushdown automaton that recognizes the reverse language of a given pushdown automaton?

            Mont Emei